LSAT and Law School Admissions Forum

Get expert LSAT preparation and law school admissions advice from PowerScore Test Preparation.

 Administrator
PowerScore Staff
  • PowerScore Staff
  • Posts: 8919
  • Joined: Feb 02, 2011
|
#23509
Complete Question Explanation

Weaken. The correct answer choice is (E)

The stimulus conclusion here is working under the assumption that aspirin's effectiveness in preventing heart attacks is immediate. If that is true, the conclusion is properly drawn. However, if the time period in which aspirin must be taken to prevent heart attacks exceeds six months, then the conclusion is flawed, given that anybody who read the study would have been unable to take the proper preventative amount in the elapsed time.

Answer Choice (A): The staff's work ethic is not the issue here. How hard the staff worked to get the study out does not change the ultimate conclusion that if they had gotten it out sooner, heart attacks would have been prevented.

Answer Choice (B): Whether the tests on laboratory animals proved ineffective is irrelevant to the issue and does not change the fact that the tests were effective on humans.

Answer Choice (C): The issue here is whether or not heart attacks could have been prevented and has nothing to do with any other ill effects. The conclusion that heart attacks could have been prevented is not affected by any side effects that may have occurred additionally.

Answer Choice (D): This very well may be true, given that it took the journal six months to publish the article. While such a policy may have many benefits and may avoid potential medical issues, it does not change the fact that the delayed publishing may have delayed the benefits to the public.

Answer Choice (E): This is the correct answer choice. Given that it takes two years of use for aspirin to have any beneficial effects, it follows that no heart attacks would have been prevented during the six week period prior to publishing.
 mariahenain
  • Posts: 30
  • Joined: Jun 09, 2017
|
#38225
I narrowed my answers to D and E and was torn between the two. I'm just not understanding why D is incorrect and E is correct?
My prephrase was: "I need to find an answer that makes it less likely that releasing the study sooner would have prevented the heart attacks. It may have had no effect."
Was I far off the mark? If so, how do you suggest sharpening prephrasing skills for weaken/strengthen/assumption?
 Francis O'Rourke
PowerScore Staff
  • PowerScore Staff
  • Posts: 471
  • Joined: Mar 10, 2017
|
#38431
In this stimulus, the speaker argued that 'If the results had been published sooner, many of the heart attacks could have been prevented.' Since this is a weaken question, your prephrase should direct you towards something along the lines of "Even if the results had been published sooner, many of the heart attacks would not have been prevented." If you are unsure, stick as close to the language of the conclusion as possible.

It sounds like your prephrase was pretty good! However, it looks like you misinterpreted answer choice (D). If we learn that the medical journal has an official extensive review policy, we would not know the effects of the policy. As the administrator said, knowing that we would have had to violate a review policy would not necessarily mean that fewer of the heart attacks would have occurred. It is possible that you thought the review policy added something helpful to the results, but answer choice (D) does not provide that information. For all we know, the review policy left the report as exactly as originally written! :-D
 Blueballoon5%
  • Posts: 156
  • Joined: Jul 13, 2015
|
#47939
Hello! I understand why answer choice E is right, but how is answer choice A wrong? I ultimately chose answer choice A because I thought: if the medical journal tried to publish the study as soon as possible, there was no way for the study to be published earlier. Since it couldn't be published earlier, there was no way that heart attacks could have been prevented during the delay.

For example, let's say that I work at a fast food restaurant. A woman enters the store and is really thirsty. I do my very best to get her the drink as soon as possible. It took about two minutes to get her order, receive the money, and get her drink. It is true that her thirst during the two minutes could have been prevented if I just gave her the drink without taking her order through the cashier (thus reducing the two minutes). However, that is not how things run in the store. I need to make the order and get the money first. Nonetheless, even with these steps, I did my best to get her order as soon as possible (within the boundaries of the store code).

With this example, I can say the same with the stimulus of this question. Considering answer choice A, the medical journal staff did their best to publish the study as soon as possible. Nonetheless, they need to go through the process of six months to finalize arrangements to publish this article. Taking answer choice A into account, doesn't this attack the conclusion (that heart attacks could have been prevented)? I could argue that, since the staff published the study as soon as possible, the heart attacks couldn't be prevented because there is no way the study could have been published earlier.
 Brook Miscoski
PowerScore Staff
  • PowerScore Staff
  • Posts: 418
  • Joined: Sep 13, 2018
|
#62674
Blueballoon,

The reason A is wrong is that the stimulus is not concerned with whether it is possible to publish the findings sooner, only what would have happened if the findings had been published sooner. Thus (A), which only concerns feasibility, is irrelevant to evaluating the strength of the argument.

Get the most out of your LSAT Prep Plus subscription.

Analyze and track your performance with our Testing and Analytics Package.